4printable answers

55
Schweser Printable Answers - Test Management Exam 4 Test ID#: 71 Question 1 - #94699 An analyst belongs to a nationally recognized charitable organization, which requires dues for membership. The analyst has worked out a deal under which he provides money management advice in lieu of paying dues. While performing services for the organization, the analyst discovers some useful computer programs that his predecessor developed and left as the property of the organization. The analyst decides to use the computer programs in his consulting business. This action is: A) a violation of Standard I(D) concerning misconduct. B) appropriate since the analyst is technically an employee of the organization. C) a violation of Standard III(B) concerning fair dealing. Your answer: B was incorrect. The correct answer was A) a violation of Standard I(D) concerning misconduct. Since the programs are the property of the organization, the analyst can only use them for the organization. It does not matter whether the analyst is an employee or not. Personal use of the programs without permission from the charitable organization is dishonest and prohibited. This question tested from Session 1, Reading 2-I, LOS D.. Question 2 - #94398 Cynthia Abbott, a CFA charterholder, is preparing a research report on Boswell Company for her employer, Capital Asset Management. Bob Carter, president of Boswell, invites Abbott and several other analysts to visit his company and offers to pay her transportation and lodging. Abbott declines Carter’s offer but, while visiting the company, accepts a gift from Carter valued at $75. Abbott fails to disclose the gift to her supervisor at Capital when she returns. In the course of the company visit, Abbott overhears a conversation between Carter and his chief financial officer that the company’s earnings per share (EPS) are expected to be $1.10 for the next quarter. Abbott was surprised that this EPS is substantially above her initial earnings estimate of $0.70 per share. Without further investigation, Abbott decides to include the $1.10 EPS in her research report on Boswell. Using the high EPS positively affects her recommendation of Boswell. Which of the following statements about whether Abbott violated Standard V(A), Diligence and Reasonable Basis and Standard I(B), Independence and Objectivity is CORRECT? Abbott: A) did not violate Standard V(A) but she violated Standard I(B). B) violated both Standard V(A) and Standard I(B). C) violated Standard V(A) but she did not violate Standard I(B). Your answer: B was incorrect. The correct answer was C) violated Standard V(A) but she did not violate Standard I(B). 71 Back to Tes tReview 71 /online_program /t Hide Q ues tions 0

Upload: mohuna

Post on 13-Apr-2015

68 views

Category:

Documents


6 download

TRANSCRIPT

Page 1: 4Printable Answers

Schweser Printable Answers - Test Management Exam 4

Test ID#: 71

Question 1 - #94699

An analyst belongs to a nationally recognized charitable organization, which requires dues for membership. The analyst has worked out a deal under which he provides money management advice in lieu of paying dues. While performing services for the organization, the analyst discovers some useful computer programs that his predecessor developed and left as the property of the organization. The analyst decides to use the computer programs in his consulting business. This action is:

A) a violation of Standard I(D) concerning misconduct.B) appropriate since the analyst is technically an employee of the organization.C) a violation of Standard III(B) concerning fair dealing.

Your answer: B was incorrect. The correct answer was A) a violation of Standard I(D) concerning misconduct.

Since the programs are the property of the organization, the analyst can only use them for the organization. It does not matter whether the analyst is an employee or not. Personal use of the programs without permission from the charitable organization is dishonest and prohibited.

This question tested from Session 1, Reading 2-I, LOS D..

Question 2 - #94398

Cynthia Abbott, a CFA charterholder, is preparing a research report on Boswell Company for her employer, Capital Asset Management. Bob Carter, president of Boswell, invites Abbott and several other analysts to visit his company and offers to pay her transportation and lodging. Abbott declines Carter’s offer but, while visiting the company, accepts a gift from Carter valued at $75. Abbott fails to disclose the gift to her supervisor at Capital when she returns. In the course of the company visit, Abbott overhears a conversation between Carter and his chief financial officer that the company’s earnings per share (EPS) are expected to be $1.10 for the next quarter. Abbott was surprised that this EPS is substantially above her initial earnings estimate of $0.70 per share. Without further investigation, Abbott decides to include the $1.10 EPS in her research report on Boswell. Using the high EPS positively affects her recommendation of Boswell.

Which of the following statements about whether Abbott violated Standard V(A), Diligence and Reasonable Basis and Standard I(B), Independence and Objectivity is CORRECT? Abbott:

A) did not violate Standard V(A) but she violated Standard I(B).B) violated both Standard V(A) and Standard I(B).C) violated Standard V(A) but she did not violate Standard I(B).

Your answer: B was incorrect. The correct answer was C) violated Standard V(A) but she did not violate Standard I(B).

Abbott violated Standard V(A), Diligence and Reasonable Basis, because she did not have a reasonable and adequate basis to support the $1.10 EPS without further investigation. By including the $1.10 EPS in her report, she did not exercise diligence and thoroughness to ensure that any research report finding is accurate. If Abbott suspects that any information in a source is not accurate, she should refrain from relying on that information. Abbott did not violate Standard I(B), Independence and Objectivity, because the gift from Carter was merely a token item.

This question tested from Session 1, Reading 2, LOS a, b, c.

Question 3 - #94761

An investment advisor goes straight from a research seminar to a meeting with a prospective new client with whom she has never been in contact. The advisor is very excited about the information she just received in the seminar and begins showing the prospect the new ideas her firm is coming up with. This is most likely a violation of:

A) Standard III(B), Fair Dealing.B) both of these.

71 Back to Test Review71 /online_program/t Hide Questions

0

Page 2: 4Printable Answers

C) Standard III(C), Suitability. Your answer: B was correct!

It is a violation of Standard III(B) because the advisor should act first on behalf of existing clients whose needs and characteristics she already knows. It is a violation of Standard III(C) because she has never met the prospect and does not know if the new ideas are appropriate for the prospect. Thus, “both of these” is the best response.

This question tested from Session 1, Reading 2-III, LOS B..

Question 4 - #94966

An analyst has a large personal holding of a security, and he has just determined that market conditions warrant selling this security. The analyst contacts clients who have a position in the security and advises them to sell some or all of the security. After waiting 24 hours, he sells the security from his personal accounts. This is:

A) a violation of Standard VI(B), Priority of Transactions.B) a violation of Standard III(B), Fair Dealing.C) congruent with Standard VI(B), Priority of Transactions.

Your answer: B was incorrect. The correct answer was C) congruent with Standard VI(B), Priority of Transactions.

According to Standard VI(B), an analyst must give clients the first opportunity to buy or sell a security before the analyst acts on his own behalf. A 24-hour waiting period seems reasonable under the circumstances presented. The analyst seems to have a reasonable basis, and there is no reason to believe that he is violating Standard III(B) since he contacted all of the clients who have a position in the security.

This question tested from Session 1, Reading 2-VI, LOS B..

Question 5 - #94604

Jason Blackwell, CFA, works as an investment manager for Mega Capital, a large multinational brokerage firm. Mega Capital is based in a country whose applicable law is stricter than the CFA Institute Code and Standards, but does business with clients in a country whose applicable law is less strict than the Code and Standards. Blackwell decides to follow the requirements of the Code and Standards for clients in the less strict country, which is sufficient to also comply with that less-strict country’s local laws. While Blackwell is still employed at Mega, Lego Associates verbally asks Blackwell to review client portfolios during evenings and weekends for a fee. Blackwell gets written consent from his immediate supervisor at Mega to undertake this independent activity for a one-month trial basis.

Which of the following statements about Blackwell’s actions involving Standard I, Professionalism, and Standard IV(A), Loyalty is most accurate? Blackwell:

A) violated both Standard I and Standard IV(A). B) violated Standard I but did not violate Standard IV(A). C) did not violate either Standard I or Standard IV(A).

Your answer: B was correct!

Blackwell violated Standard I, Professionalism. Jameson must comply with the strictest requirements among the laws of the country where his firm is based, the CFA Institute Code and Standards, and the laws of the country where he is doing business. Because the applicable laws in Mega Capital’s home country are stricter than the Code and Standards, Jameson must additionally adhere to that more strict law.

This question tested from Session 1, Reading 2-I, LOS A..

Question 6 - #94962

During 2004 Nancy Arnold received an undergraduate business degree with a management major and completed all requirements for the CFA designation imposed by CFA Institute. She is applying for employment at several brokerage firms. Her resume states, “I was awarded the CFA degree in 2004 by CFA Institute.” Her resume also states that she graduated “with honors” and majored in finance. Her grade point average was 3.48 but “with honors” requires a 3.50 grade point average.

Page 3: 4Printable Answers

Which of the following statements about Standard VII(B), Reference to CFA Institute, the CFA Designation, and the CFA Program, and Standard I(C), Misrepresentation, is CORRECT? Arnold:

A) violated both Standard VII(B) and Standard I(C). B) violated Standard I(C) but she did not violate Standard VII(B). C) did not violate either Standard VII(B) or Standard I(C).

Your answer: B was incorrect. The correct answer was A) violated both Standard VII(B) and Standard I(C).

Arnold violated Standard VII(B). The CFA designation should not be referred to as a degree. Arnold also violated Standard I(C) because her claim that she graduated “with honors” is not true.

This question tested from Session 1, Reading 2-VII, LOS B..

Question 7 - #95012

Jane Talbot, CFA, is a portfolio manager at Cavalier Investments. Talbot manages the account of Wendall Wilcox. The performance of Wilcox's portfolio has been below that of the benchmark portfolio, the S&P 500, for the past several years. In an effort to enhance his portfolio's performance, Wilcox offers to pay Talbot $2,000 each year that his portfolio's return exceeds that of the S&P 500. Wilcox suggests this arrangement last for the next three years. The amount that Wilcox agrees to pay Talbot is in addition to the compensation that Talbot will receive from his employer and the standard fee that Wilcox will pay Cavalier for managing his portfolio over the three-year period. Talbot agrees to the arrangement proposed by Wilcox and informs Cavalier in writing of the terms of the agreement under which she will receive additional compensation. According to CFA Institute Standards of Professional Conduct Talbot must disclose:

A) the nature and amount of compensation plus the duration of the agreement.B) both the nature and amount of compensation only.C) the nature of the compensation only.

Your answer: B was incorrect. The correct answer was A) the nature and amount of compensation plus the duration of the agreement.

Procedures for compliance for Standard IV(B) indicate that the written report should state the terms of any oral or written agreement under which Talbot will receive additional compensation including the nature of the compensation, the amount of compensation and the duration of the agreement.

This question tested from Session 1, Reading 2-IV, LOS B..

Question 8 - #93527

An analyst working at an investment firm has a client that rents limousines. The client tells the analyst that as long as he is the client’s analyst, he can have free use of a limousine several times a year. The analyst needs to:

A) inform his supervisor in writing of the offer if the analyst intends to accept the offer. B) do nothing since the offer is not linked to the performance of the client's portfolio.C) explicitly refuse such an offer.

Your answer: B was incorrect. The correct answer was A) inform his supervisor in writing of the offer if the analyst intends to accept the offer.

Standard IV(B) requires that members disclose to their employer in writing all benefits that they receive in addition to their regular compensation for services they perform on behalf of their employer. They also need to get consent from their employer in writing. The written report to the employer should include the details of any written or oral agreement for extra compensation. The analyst does not have to refuse the offer.

This question tested from Session 1, Reading 2-IV, LOS B..

Question 9 - #94394

Lisa Pierce, CFA, has been researching Lander Manufacturing for the past three weeks. She likes the company’s history of fulfilling its contracts on time and within budget. She learns from the uncle of a maintenance worker at Lander’s headquarters that a group of well-dressed individuals arrived at headquarters in a lime green-colored limousine. Pierce knows from publicly available information that Gilbert Controls needs a large supply of specialized motors in its domestic

Page 4: 4Printable Answers

division. She also knows that the executive officers of Gilbert usually travel in a lime green limousine. Pierce concludes that it is very likely that Gilbert will offer a large contract to Lander. Based on this development and her prior research Pierce would like to acquire Lander Manufacturing shares for her client accounts.

Pierce should:

A) not acquire the shares because she possesses material nonpublic information.

B)not acquire the shares until after she has contacted Lander's management and encouraged them to publicly announce information about the Gilbert Controls contract. She should also wait until Lander has made the announcement and the public has had time to react to it and then make the acquisition.

C) proceed to acquire the shares.Your answer: B was incorrect. The correct answer was C) proceed to acquire the shares.

Standard II(A) prohibits members from taking investment action if they possess material nonpublic information. Pierce combined information that was not misappropriated, with her knowledge of the company, to reach a conclusion under the mosaic theory, which is permissible under the standards. She can proceed to buy the shares.

This question tested from Session 1, Reading 2-II, LOS A..

Question 10 - #93502

In the course of reviewing the Corn Co., an analyst has received comments from management that, while not meaningful by themselves, when pieced together with data he has accumulated from outside sources, lead him to recommend placing Corn Co. on his firm's sell list. What should the analyst do?

A)Show his report to his own manager and counsel for their review since this information has become material once it was combined with his analysis.

B) Not issue the report until the comments are publicly announced.

C)The comments are non material and the report can be issued as long as he maintains a file of the facts as supplied by management.

Your answer: B was incorrect. The correct answer was C) The comments are non material and the report can be issued as long as he maintains a file of the facts as supplied by management.

This is an example of the mosaic theory where separate pieces of nonmaterial information are pieced together to make an investment recommendation.

This question tested from Session 1, Reading 2, LOS a, b, c.

Question 11 - #93501

Marc Randall, CFA, is an investment analyst. During a meeting with a potential client, Randall's boss states that, "You can be sure our investments will always outperform Treasury Bonds because of our fine research staff members, like Marc." Randall knows that this statement is:

A) a violation of the Standard concerning prohibition against misrepresentation.B) a violation of fiduciary duties owed to clients under the Standards.C) not in violation of the Code and Standards.

Your answer: B was incorrect. The correct answer was A) a violation of the Standard concerning prohibition against misrepresentation.

Under Standard I(C), members are forbidden from guaranteeing a specific rate of return on volatile investments. Therefore, the statement is in violation of the Standard.

This question tested from Session 1, Reading 2-I, LOS C..

Question 12 - #94348

Jack Stevens is employed by a company to provide investment advice to participants in the firm's 401(k) plan. One of the investment options is a stable value fund run by the company. Stevens' research indicates that the fund is far riskier and less liquid than the typical stable value fund and has a fundamental asset value lower than book value of the assets. He

Page 5: 4Printable Answers

tells Jessica Cox, the head of employee benefits, about his research, and indicates that he will advise new employees to not invest in the fund and will advise employees who already own the fund to reduce their holdings in the fund. Cox points out that the fund is not in any current danger because there are very few redemptions requested of the fund. Cox also states that a sell recommendation may become a self fulfilling prophecy, causing investors to redeem their shares and forcing the fund to liquidate, which in turn will cause the remaining investors to receive less than their promised value. Stevens agrees with this assessment and feels his fiduciary duty is to all employees. Stevens should:

A) tell investors he cannot give advice on the fund because of a conflict of interest.

B)continue to recommend that new investors do not invest in the fund and existing investors reduce their holdings.

C)continue to recommend that new investors do not invest in the fund, but not advise existing investors to reduce their holdings.

Your answer: B was correct!

The employees to whom Stephens owes fiduciary duty are the ones who are seeking his advice, even if acting on that advice hurts other employees who might eventually become clients.

This question tested from Session 1, Reading 2, LOS a, b, c.

Question 13 - #93559

Personal Advisers, Inc., has determined four possible economic scenarios and has projected the portfolio returns for two portfolios for their client under each scenario. Personal’s economist has estimated the probability of each scenario as shown in the table below. Given this information, what is the covariance of the returns on Portfolio A and Portfolio B?

Scenario Probability Return on Portfolio A Return on Portfolio B A 15% 18% 19% B 20% 17% 18% C 25% 11% 10% D 40% 7% 9%

A)

0.890223.

B)

0.001898.

C)

0.002019.

Your answer: B was correct! S P (S) Return on Portfolio A RA – E(RA) Return on Portfolio B RB – E(RB) [RA – E(RA)]

x [RB – E(RB)]x P(S)

A 15% 18% 6.35% 19% 6.45% 0.000614

B 20% 17% 5.35% 18% 5.45% 0.000583

C 25% 11% –0.65% 10% –2.55% 0.000041

D 40% 7% –4.65% 9% –3.55% 0.000660

E(RA) =11.65% E(RB) =12.55% Cov(RA,RB) =0.001898

This question tested from Session 2, Reading 8, LOS k.

Question 14 - #93723

A portfolio manager wants to eliminate four stocks from a portfolio that consists of six stocks. How many ways can the four stocks be sold when the order of the sales is important?

A) 180.B) 24.C) 360.

Your answer: B was incorrect. The correct answer was C) 360.

Page 6: 4Printable Answers

This is a choose four from six problem where order is important. Thus, it requires the permutation formula: n! / (n − r)! = 6! / (6 − 4)! = 360.

With TI calculator: 6 [2nd][nPr] 4 = 360.

This question tested from Session 2, Reading 8, LOS o.

Question 15 - #93628

There is a 30% chance that the economy will be good and a 70% chance that it will be bad. If the economy is good, your returns will be 20% and if the economy is bad, your returns will be 10%. What is your expected return?

A) 13%.B) 17%.C) 15%.

Your answer: B was incorrect. The correct answer was A) 13%.

Expected value is the probability weighted average of the possible outcomes of the random variable. The expected return is: ((0.3) × (0.2)) + ((0.7) × (0.1)) = (0.06) + (0.07) = 0.13.

This question tested from Session 2, Reading 8, LOS l.

Question 16 - #93818

Twenty Level I CFA candidates in a study group took a practice exam and want to determine the distribution of their scores. When they grade their exams they discover that one of them skipped an ethics question and subsequently filled in the rest of his answers in the wrong places, leaving him with a much lower score than the rest of the group. If they include this candidate’s score, their distribution will most likely:

A) have a mode that is less than its median.B) be positively skewed.C) have a mean that is less than its median.

Your answer: B was incorrect. The correct answer was C) have a mean that is less than its median.

With the low outlier included, the distribution will be negatively skewed. For a negatively skewed distribution, the mean is less than the median, which is less than the mode.

This question tested from Session 2, Reading 7, LOS k.

Question 17 - #93709

In a skewed distribution, what is the minimum amount of observations that will fall between +/- 1.5 standard deviations from the mean?

A) 44%.B) 56%.C) 95%.

Your answer: B was correct!

Because the distribution is skewed, we must use Chebyshev’s Inequality, which states that the proportion of observations within k standard deviations of the mean is at least 1 – (1 / k2).

1 – (1 / 1.52) = 0.5555, or 56%.

This question tested from Session 2, Reading 7, LOS h.

Question 18 - #93590

Page 7: 4Printable Answers

Justin Banks just won the lottery and is trying to decide between the annual cash flow payment option or the lump sum option. He can earn 8% at the bank and the annual cash flow option is $100,000/year, beginning today for 15 years. What is the annual cash flow option worth to Banks today?

A) $855,947.87.B) $1,080,000.00.C) $924,423.70.

Your answer: B was incorrect. The correct answer was C) $924,423.70.

First put your calculator in the BGN.

N = 15; I/Y = 8; PMT = 100,000; CPT → PV = 924,423.70.

Alternatively, do not set your calculator to BGN, simply multiply the ordinary annuity (end of the period payments) answer by 1 + I/Y. You get the annuity due answer and you don’t run the risk of forgetting to reset your calculator back to the end of the period setting.

OR N = 14; I/Y = 8; PMT = 100,000; CPT → PV = 824,423.70 + 100,000 = 924,423.70.

This question tested from Session 2, Reading 5, LOS e.

Question 19 - #93962

According to the Central Limit Theorem, the distribution of the sample means is approximately normal if:

A) the underlying population is normal.B) the standard deviation of the population is known.C) the sample size n > 30.

Your answer: B was incorrect. The correct answer was C) the sample size n > 30.

The Central Limit Theorem states that if the sample size is sufficiently large (i.e. greater than 30) the sampling distribution of the sample means will be approximately normal.

This question tested from Session 3, Reading 10, LOS e.

Question 20 - #93932

The following data are available on a sample of advertising budgets of 81 U.S. manufacturing companies: The mean budget is $10 million. The sample variance is 36 million. The standard error of the sample mean is:

A) $667.B) $400.C) $1,111.

Your answer: B was incorrect. The correct answer was A) $667.

The sample standard deviation is the square root of the variance: (36,000,000)1/2 = $6,000. The standard error of the sample mean is estimated by dividing the standard deviation of the sample by the square root of the sample size: σmean = s / (n)1/2 = 6,000 / (81)1/2 = $667.

This question tested from Session 3, Reading 10, LOS f.

Question 21 - #93946

Frank Grinder is trying to introduce sampling into the quality control program of an old-line manufacturer. Grinder samples 38 items and finds that the standard deviation in size is 0.019 centimeters. What is the standard error of the sample mean?

A) 0.00308.

Page 8: 4Printable Answers

B) 0.00204.C) 0.00615.

Your answer: B was incorrect. The correct answer was A) 0.00308.

If we do not know the standard deviation of the population (in this case we do not), then we estimate the standard error of the sample mean = the standard deviation of the sample / the square root of the sample size = 0.019 / √38 = 0.00308 centimeters.

This question tested from Session 3, Reading 10, LOS f.

Question 22 - #93908

A sample of 100 individual investors has a mean portfolio value of $28,000 with a standard deviation of $4,250. The 95% confidence interval for the population mean is closest to:

A) $27,159 to $28,842.B) $19,500 to $28,333.C) $27,575 to $28,425.

Your answer: B was incorrect. The correct answer was A) $27,159 to $28,842.

Confidence interval = mean ± tc{S / √n}

= 28,000 ± (1.98) (4,250 / √100) or 27,159 to 28,842

If you use a z-statistic because of the large sample size, you get 28,000 ± (1.96) (4,250 / √100) = 27,167 to 28,833, which is closest to the correct answer.

This question tested from Session 3, Reading 10, LOS j.

Question 23 - #94091

An analyst calculates that the mean of a sample of 200 observations is 5. The analyst wants to determine whether the calculated mean, which has a standard error of the sample statistic of 1, is significantly different from 7 at the 5% level of significance. Which of the following statements is least accurate?:

A)The mean observation is significantly different from 7, because the calculated Z-statistic is less than the critical Z-statistic.

B) The alternative hypothesis would be Ha: mean > 7.C) The null hypothesis would be: H0: mean = 7.

Your answer: B was correct!

The way the question is worded, this is a two tailed test.The alternative hypothesis is not Ha: M > 7 because in a two-tailed test the alternative is =, while < and > indicate one-tailed tests. A test statistic is calculated by subtracting the hypothesized parameter from the parameter that has been estimated and dividing the difference by the standard error of the sample statistic. Here, the test statistic = (sample mean – hypothesized mean) / (standard error of the sample statistic) = (5 - 7) / (1) = -2. The calculated Z is -2, while the critical value is -1.96. The calculated test statistic of -2 falls to the left of the critical Z-statistic of -1.96, and is in the rejection region. Thus, the null hypothesis is rejected and the conclusion is that the sample mean of 5 is significantly different than 7. What the negative sign shows is that the mean is less than 7; a positive sign would indicate that the mean is more than 7. The way the null hypothesis is written, it makes no difference whether the mean is more or less than 7, just that it is not 7.

This question tested from Session 3, Reading 11, LOS c.

Question 24 - #94234

Robert Patterson, an options trader, believes that the return on options trading is higher on Mondays than on other days. In order to test his theory, he formulates a null hypothesis. Which of the following would be an appropriate null hypothesis? Returns on Mondays are:

Page 9: 4Printable Answers

A) greater than returns on other days.B) not greater than returns on other days.C) less than returns on other days.

Your answer: B was correct!

An appropriate null hypothesis is one that the researcher wants to reject. If Patterson believes that the returns on Mondays are greater than on other days, he would like to reject the hypothesis that the opposite is true–that returns on Mondays are not greater than returns on other days.

This question tested from Session 3, Reading 11, LOS a.

Question 25 - #143190

A decrease in the price of Good Y can result in a decrease of the quantity of Good Y demanded by consumers if the substitution effect:

A) and the income effect are negative.B) is negative and larger than the positive income effect.C) is positive and the income effect is negative and larger than the substitution effect.

Your answer: B was incorrect. The correct answer was C) is positive and the income effect is negative and larger than the substitution effect.

If the price of Good Y decreases, the substitution effect will have a positive impact on the quantity demanded of Good Y. Thus, the only way that quantity demanded of Good Y can decrease is if there is a negative income effect that is greater in magnitude than the substitution effect; i.e., if Good Y is a Giffen good.

This question tested from Session 4, Reading 14, LOS f.

Question 26 - #97089

Natural monopolies exist because they can produce at lower costs with greater output, which means there are economies of scale. Which of the following industries is typically a natural monopoly?

A) Utilities.B) Technology.C) Oil.

Your answer: B was incorrect. The correct answer was A) Utilities.

With a natural monopoly average costs of production will be lowest when a single large firm produces the entire output demanded such as a utility.

This question tested from Session 4, Reading 16, LOS a.

Question 27 - #97344

Consider the following statements:

Statement 1: “When oligopoly firms cheat on price fixing agreements, the resulting price and output quantity approaches that of perfect competition.”

Statement 2: “Monopolistic competition is inefficient because a large deadweight loss from advertising and marketing costs is a characteristic of this form of competition.”

With respect to these statements:

A) both are correct.B) only one is correct.C) both are incorrect.

Your answer: B was correct!

Page 10: 4Printable Answers

The efficiency of monopolistic competition is not clear. While increased opportunity cost is associated with the intensive marketing and advertising activities that are characteristic of monopolistic competition, consumers definitely benefit from these selling activities because they receive information that often enables them to make better purchasing decisions. Hence the advertising and marketing costs may be more than the efficient amount, but do not represent a deadweight loss.

This question tested from Session 4, Reading 16, LOS d.

Question 28 - #96902

Under perfect competition, the short-run market supply curve is most accurately described by which of the following statements? The market short-run supply curve is the:

A) sum of the quantities at each price along the marginal cost curves for all firms in a given industry.B) sum of the quantities at each price along the average total cost curve for all firms in a given industry.C) average of the quantities at each price along the marginal cost curve for all firms in a given industry.

Your answer: B was incorrect. The correct answer was A) sum of the quantities at each price along the marginal cost curves for all firms in a given industry.

The short-run market supply curve is the horizontal sum of the marginal cost curves for all firms in a given industry. It is the sum of all quantities from all firms at each price along each firm’s marginal cost curve.

This question tested from Session 4, Reading 16, LOS c.

Question 29 - #138321

A firm’s most likely initial response to a cyclical increase in the inventory-to-sales ratio is to adjust their utilization of labor by:

A) adding new workers.B) laying off employees.C) reducing overtime.

Your answer: B was incorrect. The correct answer was C) reducing overtime.

As a cyclical indicator, an increase in the inventory-to-sales ratio is a sign of slowing economic growth. When decreasing their utilization of labor in response to a slowing economy, firms typically first reduce overtime. Firms tend to be slow to lay off workers until it is clear that an economic contraction is underway.

This question tested from Session 5, Reading 18, LOS b.

Question 30 - #96501

When comparing a barter economy with an economy that uses money as a medium of exchange we would expect increased efficiencies due to a reduction in which of the following?

A) The need to specialize.B) Transaction costs.C) Nominal interest rates.

Your answer: B was correct!

Money functions as a medium of exchange because it is accepted as payment for goods and services. Compare this to a barter economy, where if I have goat and want an ox, I have to find someone willing to trade. Finding someone takes time and time is costly. With money, I can sell the goat and buy the ox. Thus, transaction costs are reduced. Having money as a medium of exchange would not reduce the inflation rate, interest rates, or the need to specialize in the production of those goods in which we have a comparative advantage (low opportunity cost producer).

This question tested from Session 5, Reading 19, LOS b.

Page 11: 4Printable Answers

Question 31 - #97024

When individuals are unemployed because they do not have perfect information concerning available jobs, this is:

A) structural unemployment.B) natural unemployment.C) frictional unemployment.

Your answer: B was incorrect. The correct answer was C) frictional unemployment.

Frictional unemployment exists because workers and employers do not have perfect information and must expend time and resources on search activities.

This question tested from Session 5, Reading 18, LOS d.

Question 32 - #96511

On January 3, Logan Industries deposited $1,000,000 in cash at Federal Savings Bank. No excess reserves were present at the time Logan made the deposit and the required reserve ratio is 10%. What is the maximum amount by which Federal Savings Bank can increase its lending?

A) $100,000.B) $10,000,000.C) $900,000.

Your answer: B was incorrect. The correct answer was C) $900,000.

Since there are no excess reserves present at the time that Logan deposited the money, the bank would be required to maintain $100,000 ($1,000,000 × 0.10) on reserve and would be able to loan out or increase the money supply by $900,000.

This question tested from Session 5, Reading 19, LOS b.

Question 33 - #89053

Which of the following statements about the foreign exchange market is least accurate?

A)In the spot market, currencies are traded for immediate delivery but in the forward market, contracts are made to buy and sell currencies for future delivery.

B)A foreign currency is at a forward discount if the forward rate expressed in domestic currency is below the spot rate, whereas a forward premium exists if the forward rate is above the spot rate.

C)Foreign exchange quotations can be expressed on a direct basis–the foreign currency price of the home currency–or an indirect basis–the home currency price of another currency.

Your answer: B was incorrect. The correct answer was C) Foreign exchange quotations can be expressed on a direct basis–the foreign currency price of the home currency–or an indirect basis–the home currency price of another currency.

Foreign exchange quotations can be expressed on a direct basis — the home currency price of another currency—or an indirect basis—the foreign currency price of the home currency.

This question tested from Session 6, Reading 21, LOS g.

Question 34 - #89110

The table below outlines the possible tradeoffs of producing units of cloth and corn for both Country A and Country B.

Country A Country B

Units of Cloth

Units of Corn Units of Cloth

Units of Corn

0 4 0 8

6 3 8 6

Page 12: 4Printable Answers

14 0 16 0

Which scenario best describes the effects of trade between the countries?

A) Both countries would gain if Country A traded corn for Country B’s cloth. B) Both countries would gain if Country A traded cloth for Country B’s corn. C) Country B would not gain from trade, because it has an absolute advantage in the production of both goods.

Your answer: B was correct!

If Country B devotes all of its resources to producing corn, it will make 8 units while, under the same circumstances, Country A will make 4 units. Therefore, Country B is twice as productive with respect to corn as Country A. Country B can also produce more cloth (16 units) in relation to Country A (14 units) supposing all resources are used for cloth. However, rather than being twice as productive as was the case for corn, it is only slightly more productive. Therefore, Country B has a comparative advantage in corn, and thus should specialize in its production, while Country A should specialize in the production of cloth.

This question tested from Session 6, Reading 20, LOS b.

Question 35 - #138185

The North American Free Trade Agreement (NAFTA) is most accurately described as a:

A) customs union.B) common market.C) free trade area.

Your answer: B was incorrect. The correct answer was C) free trade area.

NAFTA is a free trade area, in which the member nations remove barriers to imports and exports among themselves. In a customs union, all members adopt common trade policies with non-members. A common market goes further, removing all barriers to movement of labor and capital among members.

This question tested from Session 6, Reading 20, LOS e.

Question 36 - #104152

Assuming no changes in the prices of a representative consumption basket in two currency areas over the measurement period, changes in the nominal exchange rate:

A) can be extrapolated to calculate interest rates.B) are equal to changes in the real exchange rate.C) can be converted to the real exchange rate using interest rates.

Your answer: B was correct!

The real interest rate = the nominal interest rate × ratio of consumption basket (or index) price levels in both countries. Assuming no price changes, the real exchange rate has remained the same as the nominal interest rate during the period.

You can think of the ratio of the consumption basket (or index) price levels in two countries as the bracketed portion of the Fisher relation for two countries. Here is the Fisher relation for two countries:

Here is the ratio of the consumption basket (or index) price levels in two countries:

Page 13: 4Printable Answers

If inflation in A is 10% and inflation in B is 0%, the ratio of consumption basket (or index) price levels is 1.1. If inflation in both countries is 0%, the ratio of consumption basket (or index) price levels is 1 and the nominal interest rate = the real interest rate. If the nominal interest rate = the real interest rate, changes in the nominal exchange rate = changes in the real exchange rate.

This question tested from Session 6, Reading 21, LOS a.

Question 37 - #98155

A furniture store acquires a set of chairs for $750 cash and sells them for $1000 cash. These transactions are most likely to affect which accounts?

Purchase Sale

A) Assets only Assets, revenue, expenses, owners' equity

B) Assets only Assets and revenues only

C) Assets and expenses Assets, revenue, expenses, owners' equity

Your answer: B was incorrect. The correct answer was A)

Assets only Assets, revenue, expenses, owners' equity

The purchase will be a decrease in cash and an increase in inventory, both asset accounts. The expense is not recorded until the chairs are sold. The sale will be a decrease in inventory and an increase in cash (assets), an increase in sales (revenues), an increase in cost of goods sold (expenses), and an increase in retained earnings (owners’ equity) for the $250 profit.

This question tested from Session 7, Reading 23, LOS c.

Question 38 - #98151

Prema Singh is the bookkeeper for Octabius Industries. Singh has been asked by the CFO of Octabius to review all purchases that occurred between February 1 and February 8 to investigate an error on the receiving dock. Singh will most likely look at the:

A) initial trial balance.B) general journal.C) general ledger.

Your answer: B was correct!

Journal entries record every transaction, showing which accounts are changed by what amounts. A listing of all the journal entries in order by date is called the “general journal.”

This question tested from Session 7, Reading 23, LOS f.

Question 39 - #98197

Which of the following best describes financial reporting and financial statement analysis?

A)Financial reports assess a company’s past performance in order to draw conclusions about the company’s ability to generate cash and profits in the future.

B)The objective of financial analysis is to provide information about the financial position of an entity that is useful to a wide range of users.

C)Financial reporting refers to how companies show their financial performance and financial analysis refers to using the information to make economic decisions.

Your answer: B was incorrect. The correct answer was C) Financial reporting refers to how companies show their financial performance and financial analysis refers to using the information to make economic decisions.

Page 14: 4Printable Answers

Financial reporting refers to the way companies show their financial performance to investors, creditors, and other interested parties by preparing and presenting financial statements. The objective of financial statements, not analysis, is to provide information about the financial position, performance and changes in financial position of an entity that is useful to a wide range of users in making economic decisions. The role of financial statement analysis, not reporting, is to use the information in a company’s financial statements, along with other relevant information, to assess a company’s past performance in order to draw conclusions about the company’s ability to generate cash and profits in the future.

This question tested from Session 7, Reading 22, LOS a.

Question 40 - #97921

Goldstar Manufacturing has an accounts receivable turnover of 10.5 times, an inventory turnover of 4 times, and payables turnover of 8 times. What is Goldstar’s cash conversion cycle?

A) 80.38 days.B) 6.50 days.C) 171.64 days.

Your answer: B was incorrect. The correct answer was A) 80.38 days.

The cash conversion cycle = average receivables collection period + average inventory processing period – payables payment period. The average receivables collection period = 365 / average receivables turnover or 365 / 10.5 = 34.76. The average inventory processing period = 365 / inventory turnover or 365 / 4 = 91.25. The payables payment period = 365 / payables turnover ratio = 365 / 8 = 45.63. Putting it all together: cash conversion cycle = 34.76 + 91.25 – 45.63 = 80.38.

This question tested from Session 8, Reading 28, LOS b.

Question 41 - #98041

Stanley Corp. had 100,000 shares of common stock outstanding throughout 2004. It also had 20,000 stock options with an exercise price of $20 and another 20,000 options with an exercise price of $28. The average market price for the company's stock was $25 throughout the year. The stock closed at $30 on December 31, 2004. What are the number of shares used to calculate diluted earnings per share for the year?

A) 105,000. B) 104,000. C) 110,000.

Your answer: B was correct!

Only the stock options with an exercise price of $20 are dilutive. The additional shares of 4,000 (20,000 − [(20,000 × 20) / 25]) are added to the 100,000 common shares outstanding.

This question tested from Session 8, Reading 25, LOS h.

Question 42 - #98037

At the beginning of 20X7, Bryan’s Bakery Company purchased a secret cookie recipe for $25,000. In addition, Bryan developed a new cake recipe at a cost of $5,000. Bryan expects to use both recipes indefinitely; however, the useful (economic) life of similar recipes has been 10 years. Assuming straight-line amortization, what amount of recipe expense should Bryan report for the year ended 20X7 and what amount should Bryan report as assets related to these recipes on its balance sheet at the end of 20X7?

Recipe expense Balance sheet A) $5,000 $25,000

B) $3,000 $30,000

C) $7,500 $22,500 Your answer: B was incorrect. The correct answer was C) $7,500 $22,500

Page 15: 4Printable Answers

The recipes are intangible assets. The purchased cookie recipe is capitalized and amortized over 10 years at $2,500 per year ($25,000 cost / 10 years). Since the cake recipe was developed internally, it is expensed immediately. Thus, total expense for 20X7 is $7,500 ($2,500 amortization expense + $5,000 cake recipe expense). The balance sheet value of the purchased recipe at the end of 20X7 is $25,000 – $2,500 = $22,500.

This question tested from Session 8, Reading 26, LOS e.

Question 43 - #94611

Which of the following statements regarding capitalizing versus expensing costs is least accurate?

A) Total cash flow is higher with capitalization than expensing.B) Capitalization results in higher profitability initially.C) Cash flow from investing is higher with expensing than with capitalization.

Your answer: B was incorrect. The correct answer was A) Total cash flow is higher with capitalization than expensing.

Total cash flow is higher with capitalization than expensing is least accurate because total cash flow would be the same under both methods, not considering tax implications.

This question tested from Session 9, Reading 30, LOS a.

Question 44 - #94329

Proceeds from issuing a bond are recorded on the statement of cash flows as an inflow from:

A) financing (CFF). B) investing (CFI). C) operations (CFO).

Your answer: B was incorrect. The correct answer was A) financing (CFF).

Cash from financing (CFF) is increased by the amount of the proceeds.

This question tested from Session 9, Reading 32, LOS a.

Question 45 - #94039

Under U.S. GAAP, which of the following statements regarding the disclosure of deferred taxes in a company’s balance sheet is most accurate?

A)Current deferred tax liability, current deferred tax asset, noncurrent deferred tax liability and noncurrent deferred tax asset are each disclosed separately.

B) There should be a combined disclosure of all deferred tax assets and liablities.

C)Current deferred tax liability and noncurrent deferred tax asset are netted, resulting in the disclosure of a net noncurrent deferred tax liability or asset.

Your answer: B was incorrect. The correct answer was A) Current deferred tax liability, current deferred tax asset, noncurrent deferred tax liability and noncurrent deferred tax asset are each disclosed separately.

Under U.S. GAAP, deferred tax assets and liabilities are classified as current or noncurrent, based on the underlying asset or liability. Under IFRS, deferred tax items are classified as noncurrent.

This question tested from Session 9, Reading 31, LOS i.

Question 46 - #87627

Karl Decker, CFA, is analyzing Keystone Semiconductor to determine if the stock would be a good investment. He has determined the following:

Management owns 15 percent of the outstanding shares.

Page 16: 4Printable Answers

Internal growth targets are aggressive. In recent quarters, profit growth has been exceptionally high. The company’s debt covenants are quite lax.

All of these characteristics are positives from the perspective of an investor looking for profit growth. But Decker is concerned about pressure on management to manipulate results. Which of the following should least concern Decker?

A) Recent operating results.B) Debt covenants.C) Management’s share holdings.

Your answer: B was correct!

Aggressive growth targets and high management ownership represent incentives to manipulate earnings. Extremely high growth often goes hand in hand with financial instability. But while strict debt covenants could drive management to manipulate earnings, lax covenants give management less reason to manipulate earnings.

This question tested from Session 10, Reading 33, LOS c.

Question 47 - #97705

Sterling Company is a start-up technology firm that has been experiencing super-normal growth over the past two years. Selected common-size financial information follows:

2007 Actual % of Sales

2008 Forecast % of Sales

Sales 100% 100%

Cost of goods sold 60% 55%

Selling and administration expenses 25% 20%

Depreciation expense 10% 10%

Net income 5% 15%

Non-cash operating working capital a 20% 25% a Non-cash operating working capital = Receivables + Inventory – Payables

For the year ended 2007, Sterling reported sales of $20 million. Sterling expects that sales will increase 50% in 2008. Ignoring income taxes, what is Sterling’s forecast operating cash flow for the year ended 2008, and is this forecast likely to be as reliable as a forecast for a large, well diversified, firm operating in mature industries?

Operating cash flow Reliable forecast A) $4.0 million No

B) $4.5 million No

C) $4.0 million Yes Your answer: B was incorrect. The correct answer was A) $4.0 million No

2008 sales are expected to be $30 million ($20 million 2007 sales × 1.5) and 2008 net income is expected to be $4.5 million ($30 million 2008 sales × 15%). 2007 non-cash operating working capital was $4 million ($20 million 2007 sales × 20%) and 2008 non-cash operating working capital is expected to be $7.5 million ($30 million 2008 sales × 25%). 2008 operating cash flow is expected to be $4 million ($4.5 million 2008 net income + $3 million 2008 depreciation – $3.5 million increase in non-cash operating working capital). Forecasts for small firms, start-ups, or firms operating in volatile industries may be less reliable than a forecast for a large, well diversified, firm operating in mature industries.

This question tested from Session 10, Reading 35, LOS b.

Question 48 - #97680

At the end of 2007, Decatur Corporation reported last-in, first-out (LIFO) inventory of $20 million, cost of goods sold (COGS) of $64 million, and inventory purchases of $58 million. If the LIFO reserve was $6 million at the end of 2006 and

Page 17: 4Printable Answers

$16 million at the end of 2007, compute first-in, first-out (FIFO) inventory at the end of 2007 and FIFO COGS for the year ended 2007.

FIFO Inventory FIFO COGSA) $26 million $54 million

B) $36 million $74 million

C) $36 million $54 millionYour answer: B was incorrect. The correct answer was C) $36 million $54 million

2007 FIFO inventory was $36 million ($20 million LIFO inventory + $16 million reserve). 2007 FIFO COGS was $54 million ($64 million LIFO COGS – $10 million increase in LIFO reserve).

This question tested from Session 10, Reading 35, LOS e.

Question 49 - #97568

Which of the following actions would most likely have a positive influence on shareholder value?

A) Executive board members regularly attend the board meetings. B) Adopting a poison pill. C) Only one class of common equity has been issued.

Your answer: B was incorrect. The correct answer was C) Only one class of common equity has been issued.

Firms with dual classes of equity can have a negative effect on shareholder value as the shareholder may have inferior voting rights. Takeover measures such as poison pills, golden parachutes, and greenmail typically have a negative effect on shareholder value. Annual elections are preferred for board members as it increases accountability. Executive board members regularly attending the meetings can potentially prevent free discussion among the independent members.

This question tested from Session 11, Reading 42, LOS g.

Question 50 - #87215

Annah Korotkin is the sole proprietor of CoverMeUp, a business that designs and sews outdoor clothing for dogs. Each year, she rents a booth at the regional Pet Expo and sells only blankets. Korotkin views the Expo as primarily a marketing tool and is happy to breakeven (that is, cover her booth rental). For the last 3 years, she has sold exactly enough blankets to cover the $750 booth rental fee. This year, she decided to make all blankets for the Expo out of high-tech waterproof/breathable material that is more expensive to produce, but that she believes she can sell for a higher profit margin. Information on the two types of blankets is as follows:

Per Unit Last Year’s (Basic) Blanket This Year’s (New) Blanket

Sales Price $25 $40

Variable Cost

$20 $33

Assuming that Korotkin remains most interested in covering the booth cost (which has increased to $840), how many more or fewer blankets (new style) does she need to sell to cover the booth cost? To cover this year’s booth costs, Korotkin needs to sell:

A) 42 fewer blankets than last year.B) 30 fewer blankets than last year.C) 42 more blankets than last year.

Your answer: B was correct!

To obtain this result, we need to calculate Last Year’s Breakeven Quantity, This Year’s Breakeven Quantity, and calculate the difference.

Step 1: Determine Last Year’s (Basic Blanket) breakeven quantity:

Page 18: 4Printable Answers

QBE = (Fixed Costs) / (Sales Price per unit − Variable Cost per unit) = 750 / (25 − 20) = 150

Step 2: Determine This Year’s (New Blanket) breakeven quantity:

QBE = (Fixed Costs) / (Sales Price per unit – Variable Cost per unit) = 840 / (40 − 33) = 120

Step 3: Determine Change in Units:

Q = QThis Year – QLast Year = 120 − 150 = −30. Korotkin needs to sell 30 fewer blankets.

This question tested from Session 11, Reading 38, LOS d.

Question 51 - #98220

Nippon Post Corporation (NPC), a Japanese software development firm, has a capital structure that is comprised of 60% common equity and 40% debt. In order to finance several capital projects, NPC will raise USD1.6 million by issuing common equity and debt in proportion to its current capital structure. The debt will be issued at par with a 9% coupon and flotation costs on the equity issue will be 3.5%. NPC’s common stock is currently selling for USD21.40 per share, and its last dividend was USD1.80 and is expected to grow at 7% forever. The company’s tax rate is 40%. NPC’s WACC based on the cost of new capital is closest to:

A) 9.6%.B) 13.1%.C) 11.8%.

Your answer: B was incorrect. The correct answer was C) 11.8%.

kd = 0.09(1 – 0.4) = 0.054 = 5.4%kce = [(1.80 × 1.07) / 21.40] + 0.07 = 0.16 = 16.0%WACC = 0.6(16.0%) + 0.4(5.4%) = 11.76%Flotation costs, treated correctly, have no effect on the cost of equity component of the WACC.

This question tested from Session 11, Reading 37, LOS l.

Question 52 - #97623

Which of the following is least likely to be considered a “best practice” regarding corporate governance?

A) Use of a third party to tabulate votes and retain voting records.B) A code of ethics that is audited and improved periodically.C) Board members are limited to a six-year term.

Your answer: B was incorrect. The correct answer was C) Board members are limited to a six-year term.

Anything beyond 2- or 3-year term limits on board membership has the potential to restrict the ability for shareholders to change the composition of the board if its members are not acting in the shareholders’ best interest.

This question tested from Session 11, Reading 42, LOS g.

Question 53 - #87174

Given the following information on the annual operating results for ArtFrames, a producer of quality metal picture frames, what is the degree of operating leverage (DOL) and the degree of financial leverage (DFL)?

Sales of $3.5 million Variable Costs at 45% of sales Fixed Costs of $1.05 million Debt interest payments on $750,000 issued at par with an annual 9.0% coupon (current yield is 7.0%)

Which of the following choices is closest to the correct answer? ArtFrame’s DOL and DFL are:

Page 19: 4Printable Answers

DOL DFL A) 3.00 1.50

B) 2.20 1.50

C) 2.20 1.08Your answer: B was incorrect. The correct answer was C) 2.20 1.08

The calculations are as follows:

First, calculate the operating results:

ArtFrames Annual Operating Results

Sales $3,500,000

Variable Costs1 1,575,000

1,925,000

Fixed Costs 1,050,000

Earnings before interest and taxes (EBIT)

875,000

Interest Expense2 67,500

807,5001Variable costs = 0.45 × 3,500,000 2Interest Expense = 0.09 × 750,000

Second, calculate DOL:

DOL = (Sales – Variable Costs) / (Sales – Variable Costs – Fixed Costs) = (3,500,000 – 1,575,000) / (3,500,000 – 1,575,000 – 1,050,000) = 2.20

Third, calculate DFL:

DFL = EBIT / (EBIT – I) = 875,000 / 807,500 = 1.08

This question tested from Session 11, Reading 38, LOS b.

Question 54 - #97448

Lincoln Coal is planning a new coal mine, which will cost $430,000 to build, with the expenditure occurring next year. The mine will bring cash inflows of $200,000 annually over the subsequent seven years. It will then cost $170,000 to close down the mine over the following year. Assume all cash flows occur at the end of the year. Alternatively, Lincoln Coal may choose to sell the site today. What minimum price should Lincoln set on the property, given a 16% required rate of return?

A) $325,859. B) $376,872. C) $280,913.

Your answer: B was incorrect. The correct answer was C) $280,913.

The key to this problem is identifying this as a NPV problem even though the first cash flow will not occur until the following year. Next, the year of each cash flow must be property identified; specifically: CF0 = $0; CF1 = -430,000; CF2-8 = +$200,000; CF9 = -$170,000. One simply has to discount all of the cash flows to today at a 16% rate. NPV = $280,913.

This question tested from Session 11, Reading 36, LOS d.

Question 55 - #96598

Which of the following is least likely an indicator of a firm’s liquidity?

A) Inventory turnover.

Page 20: 4Printable Answers

B) Cash as a percentage of sales.C) Amount of credit sales.

Your answer: B was incorrect. The correct answer was C) Amount of credit sales.

No inferences about liquidity are warranted based on this measure. A firm may have higher credit sales than another simply because it has more sales overall. Cash as a proportion of sales and inventory turnover are indicators of liquidity.

This question tested from Session 11, Reading 40, LOS b.

Question 56 - #96950

Carlos Rodriquez, CFA, and Regine Davis, CFA, were recently discussing the relationships between capital structure, capital budgets, and net present value (NPV) analysis. Which of the following comments made by these two individuals is least accurate?

A)“For projects with more risk than the average firm project, NPV computations should be based on the marginal cost of capital instead of the weighted average cost of capital.”

B)“The optimal capital budget is determined by the intersection of a firm’s marginal cost of capital curve and its investment opportunity schedule.”

C) “A break point occurs at a level of capital expenditure where one of the component costs of capital increases.”Your answer: B was incorrect. The correct answer was A) “For projects with more risk than the average firm project, NPV computations should be based on the marginal cost of capital instead of the weighted average cost of capital.”

The marginal cost of capital (MCC) and the weighted average cost of capital (WACC) are the same thing. If a firm’s capital structure remains constant, the MCC (WACC) increases as additional capital is raised.

This question tested from Session 11, Reading 37, LOS c.

Question 57 - #97208

Deighton Industries has 200,000 bonds outstanding. The par value of each corporate bond is $1,000, and the current market price of the bonds is $965. Deighton also has 6 million common shares outstanding, with a book value of $35 per share and a market price of $28 per share. At a recent board of directors meeting, Deighton board members decided not to change the company’s capital structure in a material way for the future. To calculate the weighted average cost of Deighton’s capital, what weights should be assigned to debt and to equity?

Debt EquityA) 48.85% 51.15%

B) 56.55% 43.45%

C) 53.46% 46.54%Your answer: B was incorrect. The correct answer was C) 53.46% 46.54%

In order to calculate the weighted average cost of capital (WACC), market value weights should be used.

For the bonds = 200,000 × $965 = $193,000,000

For the stocks = 6,000,000 × $28 = $168,000,000

$361,000,000

The weight of debt would be: 193,000,000 / 361,000,000 = 0.5346 = 53.46%The weight of common stock would be: 168,000,000 / 361,000,000 = 0.4654 = 46.54%

This question tested from Session 11, Reading 37, LOS c.

Question 58 - #96782

The before-tax cost of debt for Hardcastle Industries, Inc. is currently 8.0%, but it will increase to 8.25% when debt levels reach $600 million. The debt-to-total assets ratio for Hardcastle is 40% and its capital structure is composed of debt and

Page 21: 4Printable Answers

common equity only. If Hardcastle changes its target capital structure to 50% debt / 50% equity, which of the following describes the effect on the level of new investment at which the cost of debt will increase? The level will:

A) change, but can either increase or decrease.B) decrease.C) increase.

Your answer: B was correct!

A break point refers to a level of new investment at which a component’s cost of capital changes. The formula for break point is:

As indicated, as the weight of a capital component in the capital structure increases, the break point at which a change in the component’s cost will decline. No computation is necessary, but when Hardcastle has 40% debt, the breakpoint is $600,000,000 / 0.4 = $1.5 billion. If Hardcastle’s debt increases to 50%, the breakpoint will decline to $600,000,000 / 0.5 = $1.2 billion.

This question tested from Session 11, Reading 37, LOS k.

Question 59 - #97509

Genoa Corp. is estimating its weighted average cost of capital (WACC). They have several pieces of data to consider. The firm pays 40% of its earnings out in dividends. The return on equity (ROE) is 15%. Last year’s earnings were $5.00 per share and the dividend was just paid to shareholders. The current price of shares is $42.00. Genoa's 8% coupon bonds have a yield to maturity of 7.5%. The firm's tax rate is 30%.

Part 1)The cost of common equity is closest to:

A) 14.19%.B) 13.76%.C) 16.14%.

Your answer: B was incorrect. The correct answer was A) 14.19%.

ROE × retention ratio = growth rate.

15% × (1 – 0.40) = 9%

D0 = $5.00 × 0.40 = $2.00

[$2.00(1.09) / $42.00] + 0.09 = 14.19%

This question tested from Session 11, Reading 37, LOS h.

Genoa Corp. is estimating its weighted average cost of capital (WACC). They have several pieces of data to consider. The firm pays 40% of its earnings out in dividends. The return on equity (ROE) is 15%. Last year’s earnings were $5.00 per share and the dividend was just paid to shareholders. The current price of shares is $42.00. Genoa's 8% coupon bonds have a yield to maturity of 7.5%. The firm's tax rate is 30%.

Part 2)The after-tax cost of debt is closest to:

A) 7.5%.B) 5.6%.C) 5.3%.

Your answer: B was incorrect. The correct answer was C) 5.3%.

7.5 × (1 − 0.3) = 5.25%

Page 22: 4Printable Answers

This question tested from Session 11, Reading 37, LOS h.

Question 60 - #93681

Which of the following statements concerning the efficient frontier is most accurate? It is the:

A) set of portfolios that gives investors the lowest risk.B) set of portfolios that gives investors the highest return.C) set of portfolios where there are no more diversification benefits.

Your answer: B was incorrect. The correct answer was C) set of portfolios where there are no more diversification benefits.

The efficient frontier outlines the set of portfolios that gives investors the highest return for a given level of risk or the lowest risk for a given level of return. It is also the point at which there are no more benefits to diversification.

This question tested from Session 12, Reading 44, LOS g.

Question 61 - #95377

If the standard deviation of stock A is 10.6%, the standard deviation of stock B is 14.6%, and the covariance between the two is 0.015476, what is the correlation coefficient?

A) 0.0002.B) 0.C) +1.

Your answer: B was incorrect. The correct answer was C) +1.

The formula is: (Covariance of A and B) / [(Standard deviation of A)(Standard Deviation of B)] = (Correlation Coefficient of A and B) = (0.015476) / [(0.106)(0.146)] = 1.

This question tested from Session 12, Reading 44, LOS c.

Question 62 - #95210

Gregg Goebel and Mason Erikson are studying for the Level I CFA examination. They have just started the section on Portfolio Management and Erikson is having difficulty with the equations for the covariance (cov1,2) and the correlation coefficient (r1,2) for two-stock portfolios. Goebel is confident with the material and creates the following quiz for Erikson. Using the information in the table below, he asks Erickson to fill in the question marks.

Portfolio J Portfolio K Portfolio L

Number of Stocks 2 2 2

Covariance ? cov1,2 = 0.020 cov1,2 = 0.003

Correlation coefficient r1,2 = 0.750 ? ?

Risk measure Stock 1 Std. Deviation1 = 0.08 Std. Deviation1 = 0.20 Std. Deviation1 = 0.18

Risk measure Stock 2 Std. Deviation2 = 0.18 Std. Deviation2 = 0.12 Variance2 = 0.09

Which of the following choices correctly gives the covariance for Portfolio J and the correlation coefficients for Portfolios K and L?

Portfolio J Portfolio K Portfolio L A) 0.011 0.002 0.076

B) 1.680 0.002 0.076

C) 0.011 0.833 0.056 Your answer: B was incorrect. The correct answer was C) 0.011 0.833 0.056

Page 23: 4Printable Answers

The calculations are as follows:

Portfolio J covariance = cov1,2 = (r1,2) × (s1) × (s2) = 0.75 × 0.08 × 0.18 = 0.0108, or 0.011.

Portfolio K correlation coefficient = (r1,2) = cov1,2 / [ (s1) × (s2) ] = 0.02 / (0.20 × 0.12) = 0.833.

Portfolio L correlation coefficient = (r1,2) = cov1,2 / [ (s1) × (s2)1/2 ] = 0.003 / (0.18 × 0.091/2) = 0.003 / (0.18 × 0.30) = 0.056.

This question tested from Session 12, Reading 44, LOS c.

Question 63 - #95240

Which of the following is least likely to be considered a constraint when preparing an investment policy statement?

A) Liquidity needs.B) Risk tolerance.C) Tax concerns.

Your answer: B was correct!

The constraints are: liquidity needs, time horizon, taxes, legal and regulatory factors, and unique needs and preferences. Risk tolerance is included in the investment objectives of the policy statement, not in the constraints.

This question tested from Session 12, Reading 46, LOS e.

Question 64 - #93377

In the context of the capital market line (CML), which of the following statements is CORRECT?

A) Firm-specific risk can be reduced through diversification.B) The two classes of risk are market risk and systematic risk.C) Market risk can be reduced through diversification.

Your answer: B was incorrect. The correct answer was A) Firm-specific risk can be reduced through diversification.

The other statements are false. Market risk cannot be reduced through diversification; market risk = systematic risk. The two classes of risk are unsystematic risk and systematic risk.

This question tested from Session 12, Reading 45, LOS c.

Question 65 - #95239

While assessing an investor’s risk tolerance, a financial adviser is least likely to ask which of the following questions?

A) “How much insurance coverage do you have?”B) “What rate of investment return do you expect?” C) “Is your home life stable?”

Your answer: B was correct!

While the degree of risk tolerance will have an affect on expected returns, assessing the risk tolerance comes first, and the resulting set of feasible returns follows. The other questions address risk tolerance.

This question tested from Session 12, Reading 46, LOS d.

Question 66 - #93495

The standard deviation of the rates of return is 0.25 for Stock J and 0.30 for Stock K. The covariance between the returns of J and K is 0.025. The correlation of the rates of return between J and K is:

Page 24: 4Printable Answers

A) 0.33.B) 0.10.C) 0.20.

Your answer: B was incorrect. The correct answer was A) 0.33.

CovJ,K = (rJ,K)(SDJ)(SDK), where r = correlation coefficient and SDx = standard deviation of stock x

Then, (rJ,K) = CovJ,K / (SDJ × SDK) = 0.025 / (0.25 × 0.30) = 0.333

This question tested from Session 12, Reading 44, LOS c.

Question 67 - #93513

Which of the following statements best describes an investment that is not on the efficient frontier?

A) There is a portfolio that has a lower return for the same risk.B) There is a portfolio that has a lower risk for the same return.C) The portfolio has a very high return.

Your answer: B was correct!

The efficient frontier outlines the set of portfolios that gives investors the highest return for a given level of risk or the lowest risk for a given level of return. Therefore, if a portfolio is not on the efficient frontier, there must be a portfolio that has lower risk for the same return. Equivalently, there must be a portfolio that produces a higher return for the same risk.

This question tested from Session 12, Reading 44, LOS g.

Question 68 - #93415

The expected rate of return is 2.5 times the 12% expected rate of return from the market. What is the beta if the risk-free rate is 6%?

A) 5.B) 4.C) 3.

Your answer: B was correct!

30 = 6 + β (12 - 6)24 = 6ββ = 4

This question tested from Session 12, Reading 45, LOS e.

Question 69 - #93394

The expected market premium is 8%, with the risk-free rate at 7%. What is the expected rate of return on a stock with a beta of 1.3?

A) 16.3%.B) 10.4%.C) 17.4%.

Your answer: B was incorrect. The correct answer was C) 17.4%.

RRStock = Rf + (RMarket − Rf) × BetaStock, where RR = required return, R = return, and Rf = risk-free rate, and (RMarket − Rf) = market premiumHere, RRStock = 7 + (8 × 1.3) = 7 + 10.4 = 17.4%.

This question tested from Session 12, Reading 45, LOS g.

Page 25: 4Printable Answers

Question 70 - #93487

An analyst gathered the following data for Stock A and Stock B:

Time Period Stock A Returns Stock B Returns

1 10% 15%

2 6% 9%

3 8% 12%

What is the covariance for this portfolio? A) 12.B) 6.C) 3.

Your answer: B was correct!

The formula for the covariance for historical data is:

cov1,2 = {Σ[(Rstock A − Mean RA)(Rstock B − Mean RB)]} / (n − 1)

Mean RA = (10 + 6 + 8) / 3 = 8, Mean RB = (15 + 9 + 12) / 3 = 12

Here, cov1,2 = [(10 − 8)(15 − 12) + (6 − 8)(9 − 12) + (8 − 8)(12 − 12)] / 2 = 6

This question tested from Session 12, Reading 44, LOS c.

Question 71 - #95710

Which of the following statements about portfolio diversification is CORRECT?

A) The efficient frontier represents individual securities.B) As the correlation coefficient moves from +1 to zero, the potential for diversification diminishes.

C)When a risk-averse investor is confronted with two investment opportunities having the same expected return, the investor will take the opportunity with the lower risk.

Your answer: B was incorrect. The correct answer was C) When a risk-averse investor is confronted with two investment opportunities having the same expected return, the investor will take the opportunity with the lower risk.

The other statements are false. The lower the correlation coefficient; the greater the potential for diversification. Efficient portfolios lie on the efficient frontier.

This question tested from Session 12, Reading 44, LOS h.

Question 72 - #97401

An investor sold a stock short and is worried about rising prices. To protect himself from rising prices he would place a:

A) stop order to buy.B) limit order to buy.C) stop order to sell.

Your answer: B was incorrect. The correct answer was A) stop order to buy.

A limit order to buy is placed below the current market price.

A limit order to sell is placed above the current market price.

A stop (loss) order to buy is placed above the current market price.

A stop (loss) order to sell is placed below the current market price.

Page 26: 4Printable Answers

A stop order becomes a market order if the price is hit.

This question tested from Session 13, Reading 47, LOS g.

Question 73 - #98262

Which of the following is NOT a reason bond market indexes are more difficult to create than stock market indexes?

A) The universe of bonds is much broader than that of stocks.B) There is a lack of continuous trade data available for bonds.C) Bond deviations tend to be relatively constant.

Your answer: B was incorrect. The correct answer was C) Bond deviations tend to be relatively constant.

Bond prices are quite volatile as measured by the bond’s duration.

This question tested from Session 13, Reading 48, LOS i.

Question 74 - #97079

A short seller:

A) often also places a stop loss sell order.B) loses if the price of the stock sold short decreases.C) does not receive the dividends.

Your answer: B was incorrect. The correct answer was C) does not receive the dividends.

The short seller pays all dividends to the lender, loses if stock prices rise, and is required to post a margin account. A short seller often places a stop buy order to protect the short sale position from a rising market.

This question tested from Session 13, Reading 47, LOS e.

Question 75 - #98143

Which of the following statements concerning market efficiency is least accurate?

A)If weak-form market efficiency holds, technical analysis cannot be used to earn abnormal returns over the long-run.

B) Market efficiency assumes that individual market participants correctly estimate asset prices.C) Tests of the semi-strong form of the EMH require that security returns be risk-adjusted using a market model.

Your answer: B was correct!

Market efficiency does not assume that individual market participants correctly estimate asset prices, but does assume that their estimates are unbiased. That is, some agents will over-estimate and some will under-estimate, but they will be correct, on average.

This question tested from Session 13, Reading 49, LOS e.

Question 76 - #98233

The table below lists information on price per share and shares outstanding for three companies–Lair Enterprises, Kurlew, Inc., and Mowe, Ltd.

As of Beginning of Year As of End of Year

Stock Price Per Share

($) # Shares Outstanding Price Per Share ($) # Shares Outstanding

Lair 15 10,000 10 10,000

Page 27: 4Printable Answers

Kurlew 45 5,000 60 5,000

Mowe 90 500 110 500

Part 1)Assume that at the beginning of the year, the value of the market-weighted index was 100. The one-year return on the market-cap weighted index is closest to:

A) 30.0%.B) 13.33%.C) 8.33%.

Your answer: B was incorrect. The correct answer was C) 8.33%.

Expand the table as follows:

As of Beginning of Year 1 As of End of Year 1

Stock Price Per

Share (in $)

# Shares Outstanding

Market Capitalization (in $)

Price Per Share (in $)

# Shares Outstanding

Market Capitalization (in $)

Lair 15 10,000 150,000 10 10,000 100,000

Kurlew 45 5,000 225,000 60 5,000 300,000

Mowe 90 500 45,000 110 500 55,000

Total 150 420,000 170 455,000

First, we will calculate the year-end market-cap weighted index value, then we will calculate the return percentage.

Value of market-cap weighted index = [(market capitalizationyear-end) / (market capitalizationbeginning of year)] × Beginning index value= (455,000 / 420,000) × 100 = 108.33

One-Year Return = [(Index valueyear-end / Index valuebeginning of year) − 1] × 100= [(108.33 / 100) − 1] × 100 = 8.33%.

This question tested from Session 13, Reading 48, LOS e.

The table below lists information on price per share and shares outstanding for three companies–Lair Enterprises, Kurlew, Inc., and Mowe, Ltd.

As of Beginning of Year As of End of Year

Stock Price Per Share

($) # Shares Outstanding Price Per Share ($) # Shares Outstanding

Lair 15 10,000 10 10,000

Kurlew 45 5,000 60 5,000

Mowe 90 500 110 500

Part 2)If the stocks in the table above are used to create a stock market index, it is least likely that:

A)an investor creating a price-weighted index using these three stocks would need to rebalance his portfolio at year-end to reflect the price changes.

B) a price-weighted index will have a downward bias compared to a value-weighted index.

C)a 5% change in the price of Kurlew would have a greater impact on a value-weighted index than a 5% change in the prices of either Lair or Mowe.

Your answer: B was incorrect. The correct answer was A) an investor creating a price-weighted index using these three stocks would need to rebalance his portfolio at year-end to reflect the price changes.

A price-weighted index assumes that the investor holds an equal number of shares of each stock in the index. Since the number of stocks did not change, the investor would not need to change his holdings.

Page 28: 4Printable Answers

The other statements are true. A market value weighted index is most influenced by the stock with the largest market capitalization (Kurlew) and does not need to be adjusted for stock splits. A price-weighted index has a built-in downward bias because of the impact of stock splits. After a stock split, the denominator is adjusted downward to keep the index at the same level as before the split. Since high-growth companies tend to announce stock splits more frequently than low-growth companies, the larger, more successful firms lose influence on the index.

This question tested from Session 13, Reading 48, LOS e.

Question 77 - #96230

Which of the following statements about the constant growth dividend discount model (DDM) is least accurate?

A) The constant growth DDM is used primarily for stable mature stocks.B) For the constant growth DDM to work, the growth rate must exceed the required return on equity.C) In the constant growth DDM dividends are assumed to grow at a constant rate forever.

Your answer: B was correct!

Dividends grow at constant rate forever.

Constant growth DDM is used for mature firms.

k must be greater than g.

This question tested from Session 14, Reading 52, LOS e.

Question 78 - #96231

According to the earnings multiplier model, which of the following factors is the least important in estimating a stock’s price-to-earnings ratio? The:

A) estimated required rate of return on the stock.B) expected dividend payout ratio.C) historical dividend payout ratio.

Your answer: B was incorrect. The correct answer was C) historical dividend payout ratio.

P/E = (D1/E1)/(k - g)

where:D1/E1 = the expected dividend payout ratiok = estimated required rate of return on the stockg = expected growth rate of dividends for the stock

The P/E is most sensitive to movements in the denominator.

This question tested from Session 14, Reading 52, LOS h.

Question 79 - #96234

An analyst gathered the following data:

An earnings retention rate of 40%. An ROE of 12%. The stock's beta is 1.2. The nominal risk free rate is 6%. The expected market return is 11%.

Assuming next year's earnings will be $4 per share, the stock’s current value is closest to:

Page 29: 4Printable Answers

A) $26.67.B) $33.32.C) $45.45.

Your answer: B was correct!

Dividend payout = 1 − earnings retention rate = 1 − 0.4 = 0.6

RS = Rf + β(RM − Rf) = 0.06 + 1.2(0.11 − 0.06) = 0.12

g = (retention rate)(ROE) = (0.4)(0.12) = 0.048

D1 = E1 × payout ratio = $4.00 × 0.60 = $2.40

Price = D1 / (k – g) = $2.40 / (0.12 – 0.048) = $33.32

This question tested from Session 14, Reading 52, LOS h.

Question 80 - #96343

Assume that the expected dividend growth rate (g) for a firm decreased from 5% to zero. Further, assume that the firm's cost of equity (k) and dividend payout ratio will maintain their historic levels. The firm's P/E ratio will most likely:

A) become undefined.B) increase.C) decrease.

Your answer: B was incorrect. The correct answer was C) decrease.

The P/E ratio may be defined as: Payout ratio / (k - g), so if k is constant and g goes to zero, the P/E will decrease.

This question tested from Session 14, Reading 52, LOS h.

Question 81 - #96357

If a preferred stock that pays a $11.50 dividend is trading at $88.46, what is the market’s required rate of return for this security?

A) 13.00%.B) 11.76%.C) 7.69%.

Your answer: B was incorrect. The correct answer was A) 13.00%.

From the formula: ValuePreferred Stock = D / kp, we derive kp = D / ValuePreferred Stock = 11.50 / 88.46 = 0.1300, or 13.00%.

This question tested from Session 14, Reading 52, LOS d.

Question 82 - #96257

Utilizing the infinite period dividend discount model, all else held equal, if the required rate of return (Ke) decreases, the model yields a price that is:

A) reduced, due to increased spread between growth and required return.B) reduced, due to the reduction in discount rate.C) increased, due to a smaller spread between required return and growth.

Your answer: B was incorrect. The correct answer was C) increased, due to a smaller spread between required return and growth.

The denominator of the single-stage DDM is the spread between required return Ke, and expected growth rate, g. The smaller the denominator, all else held equal, the larger the computed value.

Page 30: 4Printable Answers

This question tested from Session 14, Reading 52, LOS e.

Question 83 - #97164

Which of the following statements concerning asset-backed securities (ABSs) is least accurate?

A) ABSs typically have lower debt ratings than the firm's other borrowings.B) The asset-backed pool may be overcollateralized to provide a credit enhancement.C) Typical assets to securitize are auto loans and credit card receivables.

Your answer: B was incorrect. The correct answer was A) ABSs typically have lower debt ratings than the firm's other borrowings.

The objective of the firm with an ABS issue typically is to get a higher debt rating (a lower cost of borrowing). Typically, the ABS has a higher debt rating, perhaps because of credit enhancements.

This question tested from Session 15, Reading 55, LOS i.

Question 84 - #96972

The concept of spot and forward rates is most closely associated with which of the following explanations of the term structure of interest rates?

A) Expectations hypothesis.B) Segmented market theory.C) Liquidity premium theory.

Your answer: B was incorrect. The correct answer was A) Expectations hypothesis.

The pure expectations theory purports that forward rates are solely a function of expected future spot rates. In other words, long-term interest rates equal the mean of future expected short-term rates. This implies that an investor could earn the same return by investing in a 1-year bond or by sequentially investing in two 6-month bonds. The implications for the shape of the yield curve under the pure expectations theory are:

If the yield-curve is upward sloping, short-term rates are expected to rise. If the curve is downward sloping, short-term rates are expected to fall. A flat yield curve implies that the market expects short-term rates to remain constant.

This question tested from Session 15, Reading 56, LOS b.

Question 85 - #96579

With an option-free zero-coupon bond the effective duration is:

A) approximately equal to its years to maturity.B) unrelated to its time to maturity.C) approximately equal to the number of semiannual periods to maturity.

Your answer: B was incorrect. The correct answer was A) approximately equal to its years to maturity.

For an option-free zero coupon bond, effective and modified duration will be almost identical and both will be approximately equal to the bond's years to maturity.

This question tested from Session 15, Reading 54, LOS f.

Question 86 - #97488

A municipal bond selling at 12% above par offers a yield of 3.2%. A taxable Treasury note selling at an 8% discount offers a yield of 4.6%. An investor in the 32.5% tax bracket wishes to purchase an equal dollar amount of both bonds. The after-tax yield of the two-bond portfolio is closest to:

Page 31: 4Printable Answers

A) 4.67%.B) 2.63%.C) 3.15%.

Your answer: B was incorrect. The correct answer was C) 3.15%.

The after-tax yield of the Treasury note is the stated yield times one minus the tax rate, or 4.6% times 67.5%, or 3.1%. To calculate the portfolio yield, take the average after-tax yields of both bonds, which is 3.15%.

This question tested from Session 15, Reading 56, LOS i.

Question 87 - #97359

External credit enhancement least likely includes:

A) corporate guarantee.B) revenue fund.C) bond insurance.

Your answer: B was correct!

External enhancements include corporate guarantees and bond insurance. A revenue fund is not an external enhancement it is an internal enhancement.

This question tested from Session 15, Reading 55, LOS i.

Question 88 - #98192

Which of the following refers to the U.S. Treasury bonds that are sold in the form of zero-coupon securities?

A) Treasury calls.B) Strip-Ts.C) Pass-throughs.

Your answer: B was correct!

The U.S. Treasury does not issue zero coupon notes and bonds, therefore investment bankers began stripping the coupons from Treasuries to create synthetic zeros to meet investor demand. The Separate Trading of Registered Interest and Principal Securities (STRIP) was introduced in 1985 to meet this need.

This question tested from Session 15, Reading 55, LOS c.

Question 89 - #95979

Anne Warner wants to buy zero-coupon bonds in order to protect herself from reinvestment risk. She plans to hold the bonds for fifteen years and requires a rate of return of 9.5%. Fifteen-year Treasuries are currently yielding 4.5%. If interest is compounded semiannually, the price Warner is willing to pay for each $1,000 par value zero-coupon bond is closest to:

A) $256.B) $498.C) $249.

Your answer: B was incorrect. The correct answer was C) $249.

Note that because the question asks for how much Warner is willing to pay, we will want to use her required rate of return in the calculation.

N = 15 × 2 = 30, FV = $1,000, I/Y = 9.5 / 2 = 4.75, PMT = 0; CPT → PV = -248.53.

The difference between the bond’s price of $249 that Warner would be willing to pay and the par value of $1,000 reflects the amount of interest she would earn over the fifteen year horizon.

Page 32: 4Printable Answers

This question tested from Session 16, Reading 57, LOS e.

Question 90 - #95932

If an investor purchases a 8 1/2s 2001 Feb. $10,000 par Treasury Note at 105:16 and holds it for exactly one year, what is the rate of return if the selling price is 105:16?

A) 8.50%.B) 8.00%.C) 8.06%.

Your answer: B was incorrect. The correct answer was C) 8.06%.

Purchase Price = [(105 + 16/32)/100] x 10,000 = $10,550.00

Selling price = [(105 + 16/32)/100] x 10,000 = $10,550.00

Interest = 8 1/2% of 10,000 = $850.00

Return = (Pend - Pbeg + Interest)/Pbeg = (10,550.00 - 10,550.00 + 850.00)/10,550.00 = 8.06%

This question tested from Session 16, Reading 57, LOS c.

Question 91 - #95828

June Klein, CFA, manages a $200 million (market value) U.S. government bond portfolio for a large institution. Klein anticipates a small, parallel shift in the yield curve of 10 basis points and wants to fully hedge the portfolio against any such change. Klein would like to use the T-bond futures contract to implement the hedge. She tabulates some essential information about her portfolio and the corresponding futures contract. The results are shown in Table 1.

Table 1: Portfolio and Treasury Bond Futures Contract Characteristics

Value of Portfolio: $100,000,000

Duration of Portfolio: 8.88438

Mar-00 Futures: 94.15625

Settlement Date: 02/17/00

Final Delivery Date: 03/31/00

First Delivery Date: 03/01/00

Klein is not as comfortable with the T-bond futures contract as she would like to be. Consequently, she decides to familiarize herself with the characteristics of the futures contract and its associated delivery process. She collects all of the deliverable bonds for the futures contract. This information is shown in Table 2. Klein will test her understanding using the highlighted bond in Table 2. The price value of a basis point (PVBP) are per $1 million par value.

Table 2: Treasury Bonds Deliverable for T-Bond Futures Contract

CouponMaturity or

first call datePrice(flat)

Accrued interest

YTM/YTCPVBP $ per million par

DurationConversion

factorCost of delivery

10.000% 11/15/15 133 24/32 2.5824 6.534% 1211.2284 1.1759 23.0331

Klein's broker supplies the characteristics of the Treasury bond that is currently the cheapest-to-deliver bond. These are shown in Table 3.

Table 3: Cheapest-to-Deliver Treasury Bond

CouponMaturity or

first call datePrice(flat)

Accruedinterest

YTM/YTCPVBP $ permillion par

Duration Conversion

factorCost ofdelivery

Page 33: 4Printable Answers

13.250% 11/15/17 135.4375 3.4217 9.166% 1110.0814 7.99429 1.4899 -4.8502

Part 1)Klein wants to compute the interest rate sensitivity of the highlighted bond in Table 2. She assumes that the yield increases by one basis point. How much, per $1 million par position, will the value of this bond change (to the nearest dollar)?

A) -$12.B) -$121,123.C) -$1,211.

Your answer: B was incorrect. The correct answer was C) -$1,211.

This is the price value of a basis point (PVBP) per one million dollar par as shown in Table 2.

This question tested from Session 16, Reading 59, LOS j.

June Klein, CFA, manages a $200 million (market value) U.S. government bond portfolio for a large institution. Klein anticipates a small, parallel shift in the yield curve of 10 basis points and wants to fully hedge the portfolio against any such change. Klein would like to use the T-bond futures contract to implement the hedge. She tabulates some essential information about her portfolio and the corresponding futures contract. The results are shown in Table 1.

Table 1: Portfolio and Treasury Bond Futures Contract Characteristics

Value of Portfolio: $100,000,000

Duration of Portfolio: 8.88438

Mar-00 Futures: 94.15625

Settlement Date: 02/17/00

Final Delivery Date: 03/31/00

First Delivery Date: 03/01/00

Klein is not as comfortable with the T-bond futures contract as she would like to be. Consequently, she decides to familiarize herself with the characteristics of the futures contract and its associated delivery process. She collects all of the deliverable bonds for the futures contract. This information is shown in Table 2. Klein will test her understanding using the highlighted bond in Table 2. The price value of a basis point (PVBP) are per $1 million par value.

Table 2: Treasury Bonds Deliverable for T-Bond Futures Contract

CouponMaturity or

first call datePrice(flat)

Accrued interest

YTM/YTCPVBP $ per million par

DurationConversion

factorCost of delivery

10.000% 11/15/15 133 24/32 2.5824 6.534% 1211.2284 1.1759 23.0331

Klein's broker supplies the characteristics of the Treasury bond that is currently the cheapest-to-deliver bond. These are shown in Table 3.

Table 3: Cheapest-to-Deliver Treasury Bond

CouponMaturity or

first call datePrice(flat)

Accruedinterest

YTM/YTCPVBP $ permillion par

Duration Conversion

factorCost ofdelivery

13.250% 11/15/17 135.4375 3.4217 9.166% 1110.0814 7.99429 1.4899 -4.8502

Part 2)Using the information in Table 2, Klein would like to compute the duration of the highlighted bond. Which is the closest to Klein's answer?

A) 8.88.B) 12.11.C) 9.06.

Your answer: B was incorrect. The correct answer was A) 8.88.

Page 34: 4Printable Answers

PVBP = (0.0001) × D × (price + accrued interest) × 10,000Note: The 10,000 is to convert the price to $1,000,000 par to match the PVBP units.Rearranging, D = PVBP ÷ (price + interest) = 1,211.2284 ÷ (133.75 + 2.5824) = 8.88

This question tested from Session 16, Reading 59, LOS j.

June Klein, CFA, manages a $200 million (market value) U.S. government bond portfolio for a large institution. Klein anticipates a small, parallel shift in the yield curve of 10 basis points and wants to fully hedge the portfolio against any such change. Klein would like to use the T-bond futures contract to implement the hedge. She tabulates some essential information about her portfolio and the corresponding futures contract. The results are shown in Table 1.

Table 1: Portfolio and Treasury Bond Futures Contract Characteristics

Value of Portfolio: $100,000,000

Duration of Portfolio: 8.88438

Mar-00 Futures: 94.15625

Settlement Date: 02/17/00

Final Delivery Date: 03/31/00

First Delivery Date: 03/01/00

Klein is not as comfortable with the T-bond futures contract as she would like to be. Consequently, she decides to familiarize herself with the characteristics of the futures contract and its associated delivery process. She collects all of the deliverable bonds for the futures contract. This information is shown in Table 2. Klein will test her understanding using the highlighted bond in Table 2. The price value of a basis point (PVBP) are per $1 million par value.

Table 2: Treasury Bonds Deliverable for T-Bond Futures Contract

CouponMaturity or

first call datePrice(flat)

Accrued interest

YTM/YTCPVBP $ per million par

DurationConversion

factorCost of delivery

10.000% 11/15/15 133 24/32 2.5824 6.534% 1211.2284 1.1759 23.0331

Klein's broker supplies the characteristics of the Treasury bond that is currently the cheapest-to-deliver bond. These are shown in Table 3.

Table 3: Cheapest-to-Deliver Treasury Bond

CouponMaturity or

first call datePrice(flat)

Accruedinterest

YTM/YTCPVBP $ permillion par

Duration Conversion

factorCost ofdelivery

13.250% 11/15/17 135.4375 3.4217 9.166% 1110.0814 7.99429 1.4899 -4.8502

Part 3)Klein would like to quantify the approximate value loss of her portfolio from an increase in yields according to her expectations. Using the information in Table 1 which of the following is the closest to Klein's answer?

A) -$888,438.B) -$8,884.C) -$1,211,228.

Your answer: B was incorrect. The correct answer was A) -$888,438.

Change in portfolio value = -0.001 × duration × portfolio value. Change in portfolio value = -0.001 × 8.88438 × $100,000,000 = −$888,438.

This question tested from Session 16, Reading 59, LOS j.

Question 92 - #96077

A zero-coupon bond matures three years from today, has a par value of $1,000 and a yield to maturity of 8.5% (assuming semi-annual compounding). What is the current value of this issue?

Page 35: 4Printable Answers

A) $78.29.B) $782.91.C) $779.01.

Your answer: B was incorrect. The correct answer was C) $779.01.

The value of the bond is computed as follows:

Bond Value = $1,000 / 1.04256 = $779.01.N = 6; I/Y = 4.25; PMT = 0; FV = 1,000; CPT → PV = 779.01.

This question tested from Session 16, Reading 57, LOS e.

Question 93 - #95636

Which of the following definitions involving derivatives is least accurate?

A) A call option gives the owner the right to sell the underlying good at a specific price for a specified time period.B) An option writer is the seller of an option.C) An arbitrage opportunity is the chance to make a riskless profit with no investment.

Your answer: B was incorrect. The correct answer was A) A call option gives the owner the right to sell the underlying good at a specific price for a specified time period.

A call option gives the owner the right to buy the underlying good at a specific price for a specified time period.

This question tested from Session 17, Reading 60, LOS a.

Question 94 - #95551

When one party pays a fixed rate of interest in an equity swap, which of the following is least accurate?

A) The fixed-rate receiver will never get more than the fixed rate.B) The equity-return payer will gain if the equity return is zero.

C)Unlike other swaps, in an equity swap the one-quarter-ahead payment is not known at the end of the previous quarter.

Your answer: B was incorrect. The correct answer was A) The fixed-rate receiver will never get more than the fixed rate.

If the periodic return on the equity is negative, the fixed-rate payer must pay the fixed rate plus the percentage of (negative) equity return, times the notional principal.

This question tested from Session 17, Reading 64, LOS b.

Question 95 - #95569

Consider a swap with a notional principal of $300 million, annual payments, and a 30E/360 daycount convention (every month has 30 days, a year has 360 days).

LIBORCounterparty ▬▬▬▬▬▬▬▬▬▬► Counterparty

A ◄▬▬▬▬▬▬▬▬▬▬ B7% Fixed

0 1 2LIBOR = 5.5% LIBOR = 6.5% LIBOR = 7%

Given the above diagram, which of the following statements is most accurate? At time period 2:

A) A pays B $1.5 million.B) B pays A $1.5 million.C) B pays A $21.0 million.

Your answer: B was correct!

Page 36: 4Printable Answers

The variable rate to be used at time period 2 is set at time period 1 (the arrears method). Therefore, the appropriate variable rate is 6.5%, the fixed rate is 7%, and the interest payments are netted. The fixed-rate payer, counterparty B, pays according to:

[Swap Fixed Rate – LIBORt-1][(# of days)/(360)][Notional Principal].

In this case, we have [0.07 – 0.065][360/360][$300 million] = 1.5 million.

This question tested from Session 17, Reading 64, LOS b.

Question 96 - #95543

A put option currently has an option premium of $3 and a strike price of $40. The market price of the stock is $42 at expiration. The expiration day value of the option is:

A) $0.B) $2.C) $5.

Your answer: B was incorrect. The correct answer was A) $0.

The expiration day value of the put is $0 because it is trading out-of the money.

This question tested from Session 17, Reading 63, LOS c.

Question 97 - #95572

Currency swap markets consist of transactions in:

A) both spot and forward contracts.B) the forward market only.C) spot markets only.

Your answer: B was incorrect. The correct answer was A) both spot and forward contracts.

In this explanation, Euro is used to represent foreign currency. In a currency swap, one counterparty (D) holds dollars and wants Euros. The other counterparty (E) holds Euros and wants dollars. They decide to swap their currency positions at the current spot exchange rate. The counterparties exchange the full notional principal at the onset of the swap. Then, on each settlement date, one party pays a fixed rate of interest on the foreign currency received, and the other party pays a floating rate on the dollars received. Interest payments are not netted. Generally, the variable interest rate on the dollar borrowings is determined at the beginning of the settlement period and paid at the end of the settlement period. At the conclusion of the swap, the notional currencies are again exchanged. Thus, currency swaps involved transactions in both the spot and forward (future) markets. A fixed-for-fixed currency swap is equivalent to a portfolio of foreign exchange forward contracts (both parties need to deliver currency in the future).

This question tested from Session 17, Reading 64, LOS b.

Question 98 - #95586

XYZ, Inc. has entered into a "plain-vanilla" interest rate swap on $5,000,000 notional principal. XYZ company pays a fixed rate of 8.5% on payments that occur at 180-day intervals. Platteville Investments, a swap broker, negotiates with another firm, SSP, to take the receive-fixed side of the swap. The floating rate payment is based on LIBOR (currently at 7.2%). At the time of the next payment (due in exactly 180 days), XYZ company will:

A) pay the dealer net payments of $65,000.B) pay the dealer net payments of $32,500.C) receive net payments of $32,500.

Your answer: B was correct!

The net payment formula for the fixed-rate payer is:

Page 37: 4Printable Answers

Fixed Rate Paymentt = (Swap Fixed Rate − LIBORt-1) × (# days in term / 360) × Notional Principal

If the result is positive, the fixed-rate payer owes a net payment and if the result is negative, then the fixed-rate payer receives a net inflow. Note:We are assuming a 360 day year.

Fixed Rate Payment = (0.085 − 0.072) × (180 / 360) × 5,000,000 = $32,500.

Since the result is positive, XYZ owes this amount to the dealer, who will remit to SSP.

This question tested from Session 17, Reading 64, LOS b.

Question 99 - #95509

Which of the following is an example of an arbitrage opportunity?

A) A put option on a share of stock has the same price as a call option on an identical share. B) A stock with the same price as another has a higher rate of return. C) A portfolio of two securities that will produce a certain return that is greater than the risk-free rate of interest.

Your answer: B was incorrect. The correct answer was C) A portfolio of two securities that will produce a certain return that is greater than the risk-free rate of interest.

An arbitrage opportunity exists when a combination of two securities will produce a certain payoff in the future that produces a return that is greater than the risk-free rate of interest. Borrowing at the riskless rate to purchase the position will produce a certain future amount greater than the amount required to repay the loan.

This question tested from Session 17, Reading 60, LOS d.

Question 100 - #95810

The settlement price of a deliverable forward contract at 6% on a $1 million 90-day Treasury bill would be:

A) determined by the market rates at expiration.B) $985,000.C) $940,000.

Your answer: B was correct!

Treasury bills are quoted as a discount from face value, which is annualized based on a 360 day year. (90/360) × 6% = 1.5%, so the contract price of the $1 million bill is [1 − 0.015] × 1,000,000 = $985,000.

This question tested from Session 17, Reading 61, LOS d.

Question 101 - #95415

Using put-call parity, it can be shown that a synthetic European put can be created by a portfolio that is:

A) short the stock, long the call, and short a pure discount bond that pays the exercise price at option expiration.B) short the stock, long the call, and long a pure discount bond that pays the exercise price at option expiration. C) long the stock, short the call, and short a pure discount bond that pays the exercise price at option expiration.

Your answer: B was correct!

A short position in the stock combined with a long call and lending the present value of the exercise price will replicate the payoffs on a put at option expiration.

This question tested from Session 17, Reading 63, LOS m.

Question 102 - #95607

Page 38: 4Printable Answers

Consider a U.S. commercial bank that takes in one-year certificates of deposit (CDs) in its Hong Kong branch, denominated in Hong Kong dollars, to fund three-year, fixed-rate loans the bank is making in the U.S. denominated in U.S. dollars. Why would this bank wish to enter into a currency swap? The bank faces the risk that the Hong Kong dollar:

A) increases in value against the U.S. dollar and the risk that interest rates increase in Hong Kong.B) decreases in value against the U.S. dollar and the risk that interest rates increase in Hong Kong.C) decreases in value against the U.S. dollar and the risk that interest rates decrease in Hong Kong.

Your answer: B was incorrect. The correct answer was A) increases in value against the U.S. dollar and the risk that interest rates increase in Hong Kong.

The bank faces two problems. First, if the Hong Kong dollar increases in value, it will take more U.S. dollars to repay the Hong Kong depositors. Indeed, if the Hong Kong dollar increases significantly, it may take more U.S. dollars to repay the Hong Kong depositors than the bank makes on the U.S. loan. Secondly, if the interest rate in Hong Kong rises, the bank pays more in interest on its CDs while the rate on the bank’s U.S. loans does not change. In this case, interest expense would rise and interest income would remain the same, which narrows the bank’s profits.

This question tested from Session 17, Reading 64, LOS b.

Question 103 - #95608

Which transaction would least likely be classified as an interest rate swap?

A) Receive AUD fixed, pay NZD floating. B) Receive U.S. fixed, pay U.S. commercial paper. C) Pay USD fixed, receive U.S. LIBOR.

Your answer: B was incorrect. The correct answer was A) Receive AUD fixed, pay NZD floating.

Because it involves two different currencies, this would be a currency swap.

This question tested from Session 17, Reading 64, LOS a.

Question 104 - #95290

An exchange-for-physicals, as it pertains to futures contracts:

A) is another term for delivering an asset to satisfy a futures contract.B) involves an agreement off the floor of the exchange.C) is another term for accepting delivery of an asset to satisfy a futures contract.

Your answer: B was correct!

An exchange-for-physicals involves an agreement between long and short contract holders to settle their respective obligations by delivery and purchase of an asset. It is executed off the floor of the exchange and reported to exchange officials who then cancel both positions.

This question tested from Session 17, Reading 62, LOS e.

Question 105 - #95186

Managers of hedge funds are typically compensated by:

A) an incentive fee, paid only if performance exceeds a “high water mark”. B) a management fee, based on the net change in value of the assets during the year.

C)a base management fee, based on the value of assets under management, plus an incentive fee, based on profits.

Your answer: B was incorrect. The correct answer was C) a base management fee, based on the value of assets under management, plus an incentive fee, based on profits.

Typical arrangements pay the manager a base fee, usually around 1% of assets, plus an incentive fee proportional to profits.

Page 39: 4Printable Answers

This question tested from Session 18, Reading 66, LOS j.

Question 106 - #95072

Money managers and individual investors can indirectly participate in the commodities market through all of the following investment vehicles EXCEPT:

A) trading commodities in small denominations.B) futures contracts.C) stocks of companies producing a commodity.

Your answer: B was incorrect. The correct answer was A) trading commodities in small denominations.

Trading the commodities themselves is direct participation. Investors can participate indirectly though futures contracts or certain commodity-linked equities.

This question tested from Session 18, Reading 66, LOS q.

Question 107 - #95093

Which of the following is least likely a disadvantage of the cost approach method of estimating the market value for real estate?

A) market value of a property may differ significantly from its construction cost. B) estimating the value of the land may be difficult.C) the replacement cost of existing improvements may be difficult to determine.

Your answer: B was incorrect. The correct answer was C) the replacement cost of existing improvements may be difficult to determine.

The market value may be more or less than what it would cost to rebuild or replace it. Estimating the value of the land portion of a property with improvements is a difficult process. The replacement cost is usually easy to determine, although it may or may not reflect the value of the improvements.

This question tested from Session 18, Reading 66, LOS f.

Question 108 - #95069

A real estate analysis estimates the market value of an income-producing property at $2,560,000. The annual gross potential rental income is $596,000, the annual property operating expenses and taxes are $178,800, and the annual vacancy and collection losses are $89,400. What capitalization rate was used by the analysis to assess the property at $2,560,000?

A) 0.1280.B) 0.1275.C) 0.1290.

Your answer: B was incorrect. The correct answer was A) 0.1280.

MV =NOI

CAP

CAP =NOI

MV

596,000 − 178,800 − 89,400= 0.128

2,560,000

This question tested from Session 18, Reading 66, LOS g.

Question 109 - #95141

A manufacturing company would seek mezzanine financing in which of the following scenarios?

Page 40: 4Printable Answers

A) A company already producing and selling a product, seeking an initial expansion of operations. B) A company preparing for an initial public offering. C) A company ready for a major marketing campaign.

Your answer: B was correct!

All of the above scenarios are different stages of later-stage financing. A company ready for a major marketing campaign or a physical plant expansion is seeking third-stage financing. An initial expansion of operations describes second-stage financing. The capital provided to prepare for an initial public offering is at the mezzanine stage.

This question tested from Session 18, Reading 66, LOS h.

Question 110 - #95098

Which of the following statements regarding real estate valuation is CORRECT?

A) The estimated market value of a property depends upon the particular investor.

B)Each property is unique, so the investment value may be dependent upon the particular use planned for the property.

C) The most reliable real estate valuation method is the cost approach. Your answer: B was correct!

The market value is completely independent of any considerations based upon the investor or potential investor. There is not a “most reliable” valuation method – all have their advantages and disadvantages. The investment value may be dependent upon the planned use of the property—remember that market value and investment value are two different things.

This question tested from Session 18, Reading 66, LOS f.

Question 111 - #95266

Which of the following statements best describes the fund-of-funds (FOF) class of hedge funds? A fund of funds:

A) is an open-end mutual fund that primarily invests in other open-end funds.B) is open to institutional investors for the purpose of seeking arbitrage situations in hedge fund pricing.C) allows smaller investors to access the hedge funds market.

Your answer: B was incorrect. The correct answer was C) allows smaller investors to access the hedge funds market.

A FOF is a fund that invests in hedge funds. They are open to both individual and institutional investors.

This question tested from Session 18, Reading 66, LOS k.

Question 112 - #95217

Which of the following statements regarding survivorship bias in hedge funds is most accurate? Survivorship bias tends to:

A) overstate the performance and understate the volatility of hedge funds.B) overstate both the performance and volatility of hedge funds.C) understate the performance and overstate the volatility of hedge funds.

Your answer: B was incorrect. The correct answer was A) overstate the performance and understate the volatility of hedge funds.

Survivorship bias exists because only the successful hedge funds submit performance data, thus overstating performance when the index is considered to be representative of the entire hedge fund population. Likewise, stable funds tend to succeed, while more volatile funds tend to go out of business, causing the database to tend to understate volatility for hedge funds as an asset class.

This question tested from Session 18, Reading 66, LOS m.

Page 41: 4Printable Answers

Question 113 - #95031

Commodity-linked securities are most appropriate for investors who are:

A) seeking speculative profits in the commodities market.B) prohibited from owning physical assets.C) seeking investments that are negatively correlated with inflation.

Your answer: B was correct!

Commodity-linked securities are an appropriate investment for those who are prohibited from directly owning real assets such as commodities, but wish to participate in the market. Commodity-linked securities are not derivatives, though their performance is linked to some commodity price. Commodity-linked securities are positively correlated with inflation.

This question tested from Session 18, Reading 66, LOS q.

Question 114 - #95158

A portfolio manager is considering the purchase of an office building. He has identified the major characteristics of a property that affect value, and has assigned a quantitative rating to each one, based upon recent comparable sales in the area. Using a regression model, he has developed benchmark values for each characteristic, which he will use to estimate the market value of the potential investment. This method of estimating property value is best described as the:

A) sales comparison approach. B) regression price model. C) hedonic price estimation.

Your answer: B was incorrect. The correct answer was C) hedonic price estimation.

The sales comparison approach uses recent transactions to estimate a benchmark value. The regression price model is a fictitious model. The hedonic price model is a variation of the sales comparison approach, but is a more formalized, structured approach.

This question tested from Session 18, Reading 66, LOS g.

Question 115 - #95125

You are going to invest in a closed-end mutual fund and are told that the net asset value of the fund is $20.40, and the share price is $18.20. What is the discount you would receive or the premium that you would pay?

A) -0.1209.B) -0.1078.C) 0.1209.

Your answer: B was correct! 18.20 − 20.40

= -0.107820.40

This question tested from Session 18, Reading 66, LOS a.

Question 116 - #95106

Based on the following information, what is the net asset value (NAV) per share. There are currently no expenses and no load.

Cap Stock Sold $109,000Price per share $10

Stock Shares Price Book Value

A 1,051 $10 $5

Page 42: 4Printable Answers

B 2,420 $35 $29

C 1,851 $9 $8

D 900 $69 $63

A) $15.96.

B) $13.26.

C) $27.03.

Your answer: B was incorrect. The correct answer was A) $15.96.

Total number of shares sold =$109,000

= 10,900 shares$10 (per shares)

Total market value = 1,051 × 10 = 10,510

2,420 × 35 = 84,700

1,851 × 9 = 16,659

900 × 69 = 62,100

173,969

NAV =173,969

= 15.96 per share10,900

This question tested from Session 18, Reading 66, LOS a.

©2011 Kaplan Schweser. All Rights Reserved.

71 Back to Test Review 71 Hide Questions 0